Difference between revisions of "2023 WSMO Team Round Problems/Problem 2"
(Created page with "==Problem== Integers <math>W,S,M,</math> and <math>O</math> satisfy <math>W+S+M+O = 13.</math> Find the maximum possible value of <math>WSMO.</math> ==Solution==") |
|||
Line 4: | Line 4: | ||
==Solution== | ==Solution== | ||
+ | From the AM-GM inequality, we want <math>W,S,M,O</math> as close to each other as possible. So, let <math>W=S=M=3,O=4\implies WSMO = \boxed{108}.</math> | ||
+ | |||
+ | ~pinkpig |
Latest revision as of 13:50, 13 September 2025
Problem
Integers and
satisfy
Find the maximum possible value of
Solution
From the AM-GM inequality, we want as close to each other as possible. So, let
~pinkpig